GRÀCIES!!

BOTIGA ONLINE
elmagatzem.blogspot.com
LLibres d'ocasió a 1, 3, 6, 9 i 12 €

dilluns, 12 d’agost del 2013

547.- El nivell del llac

547.- Un pescador està amb la seva barca al mig d'un llac i tira l'ancora a l'aigua. Això farà pujar, baixar o deixarà igual el nivell de l'aigua del llac?

TROBAREU LA SOLUCIÓ ALS COMENTARIS. FELICITATS A " Sergi "

15 comentaris:

Sergi ha dit...

Jo diria que es queda igual perquè el pes de l'àncora ja estava a la barca, ja anava inclòs en el que la barca havia fet modificar el nivell del llac. He dit algun cop que la física era la meva bèstia negra a l'insti...?

sa lluna ha dit...

Jo diria que el fa pujar, però sent un llac poc li farà una àncora, crec.

Molt bona tarda, Mac!!
Aferradetes

Lluna ha dit...

A mi tb em sembla que l'hauria de fer pujar, encara que el que diu en XeXu sembla correcte tb ;)

Bona tarda XeXu!!!
Bona tarda sa lluna!!!
Bona tarda Mc!!!

Sergi ha dit...

Com que ja han dit que es quedarà igual i que pujarà, jo dic que baixa. A més crec que es la resposta correcta, ja que quan l'ancora estava al vaixell el seu pes feia que la barca desplacés un pes d'aigua igual cap amunt. En canvi, un cop dins de l'aigua, desplaça aigua cap amunt equivalent al seu volum.

Com que l'àncora és més densa que l'aigua, desplaçarà més litres cap amunt mitjançant el seu pes que no pas el seu volum.

Sergi ha dit...

Ja sabia jo que si parlàvem de física tenia les de perdre... la densitat...

Garbí24 ha dit...

l'ancora pesa menys dins l'aigua per tant desplaça menys aigua i el nivell baixa.....

Assumpta ha dit...

Doncs, clar... ehem... el que jo pensava... que... ehem... que sí, que baixa, baixa :-DDD

Drapaire de mots ha dit...

Uf, com vaig de física! quan he llegit en XeXu, he pensat que tenia raó, però després de llegir en Sergi i en Joan... definitivament qui m'ho ha acabat d'aclarir sens dubte a estat l'Assumpta. Jo = que ella plash,plash!!

jpmerch ha dit...

Hauríem de saber si l'ancora flota o no.

Jo crec que el nivell quedaria igual, però a aquestes hores, no veig la forma de demostrar-ho. Potser demá.

Consol ha dit...

Com ja han dit, Crec que el nivell de l'aigua baixarà. L'àncora té una densitat més gran que l'aigua i no flota, sinó no seria una àncora. Quan traiem aquest pes de la barca, aquesta flota més (la línia de flotació de la barca és més avall) i per tant, desplaça menys aigua.

Elfreelang ha dit...

jo diria que es quedarà igual ...però la veritat no ho sé

McAbeu ha dit...

Segons el LLIBRE: Per solucionar l'enigma s'ha d'aplicar el Principi d'Arquimedes, quan l'àncora és dins de la barca desallotjarà una quantitat d'aigua equivalent al seu pes però quan el pescador l'introdueix al llac ho farà en equivalència al seu volum. Com que es tracta d'un objecte pesat i dens, l'aigua desallotjada pel volum serà menor que la del pes i per tant el nivell del llac baixarà. Felicitats doncs a SERGI que és el primer que ens dóna la solució correcta. :-))

PS: Si, com diu JPMERCH, l'àncora flotés aleshores l'explicació aniria al revés i l'aigua del llac pujaria. Però, com molt bé apunta CONSOL, una àncora que floti no es pot considerar una àncora "com cal". :-D

Lluna ha dit...

Sort que ara per ara la majoria d'àncores són "com cal" :P

Assumpta ha dit...

Hahahaha em moro de riure imaginant això de tirar l'àncora i que floti!!

Sabeu que l'exèrcit espanyol té uns submarins que no floten? sí, sí, ja sé que els submarins són per anar per sota... però després bé han de tornar a pujar, no? :-P

McAbeu ha dit...

LLUNA i ASSUMPTA: Tal com estan les coses, avui en dia no ens en podem fiar de res... ni de les àncores! :-DD
I pel que fa a l'exèrcit espanyol, ja fa molts anys que vaig fer la mili però jo diria que segueix sent tan poc de fiar com sempre. Una impressió purament personal però que sembla quedar confirmada per aquesta notícia que ens expliques, ASSUMPTA. :-))

Publica un comentari a l'entrada